Extending Rolle's Theorem to Infinity

The name of the pictureThe name of the pictureThe name of the pictureClash Royale CLAN TAG#URR8PPP











up vote
1
down vote

favorite












Let $f : mathbbR to mathbbR$ be a function which is differentiable everywhere on $mathbbR$. If we know that
$$lim_xto-inftyf(x) = L = lim_xtoinftyf(x)$$
How would one go about showing that there must exist $c in mathbbR$, such that $f'(c) = 0$ for all cases at once?










share|cite|improve this question

























    up vote
    1
    down vote

    favorite












    Let $f : mathbbR to mathbbR$ be a function which is differentiable everywhere on $mathbbR$. If we know that
    $$lim_xto-inftyf(x) = L = lim_xtoinftyf(x)$$
    How would one go about showing that there must exist $c in mathbbR$, such that $f'(c) = 0$ for all cases at once?










    share|cite|improve this question























      up vote
      1
      down vote

      favorite









      up vote
      1
      down vote

      favorite











      Let $f : mathbbR to mathbbR$ be a function which is differentiable everywhere on $mathbbR$. If we know that
      $$lim_xto-inftyf(x) = L = lim_xtoinftyf(x)$$
      How would one go about showing that there must exist $c in mathbbR$, such that $f'(c) = 0$ for all cases at once?










      share|cite|improve this question













      Let $f : mathbbR to mathbbR$ be a function which is differentiable everywhere on $mathbbR$. If we know that
      $$lim_xto-inftyf(x) = L = lim_xtoinftyf(x)$$
      How would one go about showing that there must exist $c in mathbbR$, such that $f'(c) = 0$ for all cases at once?







      calculus real-analysis






      share|cite|improve this question













      share|cite|improve this question











      share|cite|improve this question




      share|cite|improve this question










      asked 5 hours ago









      Eetu Koskela

      206




      206




















          3 Answers
          3






          active

          oldest

          votes

















          up vote
          4
          down vote













          Consider
          $$
          g(x)=f(tan x)
          $$

          defined over $(-pi/2,pi/2)$. Then we can extend it to $[-pi/2,pi/2]$ by setting
          $$
          g(-pi/2)=g(pi/2)=L
          $$

          The derivative of $g$ exists over $(-pi/2,pi/2)$ and is equal to
          $$
          g'(x)=fracf'(tan x)cos^2x
          $$

          By Rolle's theorem, there exists $c$ such that $g'(c)=0$, so that $f'(tan c)=0$.



          You can do similarly for a function $f$ defined over $[a,infty)$, differentiable over $(a,infty)$ and such that
          $$
          f(a)=lim_xtoinftyf(x)
          $$

          Define $g(x)=f(tan x)$ over $[arctan a,pi/2)$ and the proof will be the same as before.






          share|cite|improve this answer



























            up vote
            3
            down vote













            If $f=L$ everywhere, the conclusion is clear. So let's assume $f(x_0)ne L$ for some $x_0.$ WLOG, $f(x_0)>L.$ Because of the given limit conditions, there exists $a>|x_0|$ such that both $f(-a),f(a) < f(x_0).$ Then the maximum value of $f$ on $[-a,a]$ can't occur at either of the end points. It thus occurs at an interior point $c,$ and at that $c$ we have $ f'(c)=0.$






            share|cite|improve this answer



























              up vote
              1
              down vote













              Derivatives satisfy the intermediate value property.



              If $f'(c) not= 0$ for all $c$, then either $f'(c) > 0$ for all $c$ or $f'(c) < 0$ for all $c$.



              In either case, $f$ is strictly monotone ruling out $displaystyle lim_x to -infty f(x) = lim_x to infty f(x)$.






              share|cite|improve this answer




















                Your Answer





                StackExchange.ifUsing("editor", function ()
                return StackExchange.using("mathjaxEditing", function ()
                StackExchange.MarkdownEditor.creationCallbacks.add(function (editor, postfix)
                StackExchange.mathjaxEditing.prepareWmdForMathJax(editor, postfix, [["$", "$"], ["\\(","\\)"]]);
                );
                );
                , "mathjax-editing");

                StackExchange.ready(function()
                var channelOptions =
                tags: "".split(" "),
                id: "69"
                ;
                initTagRenderer("".split(" "), "".split(" "), channelOptions);

                StackExchange.using("externalEditor", function()
                // Have to fire editor after snippets, if snippets enabled
                if (StackExchange.settings.snippets.snippetsEnabled)
                StackExchange.using("snippets", function()
                createEditor();
                );

                else
                createEditor();

                );

                function createEditor()
                StackExchange.prepareEditor(
                heartbeatType: 'answer',
                convertImagesToLinks: true,
                noModals: true,
                showLowRepImageUploadWarning: true,
                reputationToPostImages: 10,
                bindNavPrevention: true,
                postfix: "",
                imageUploader:
                brandingHtml: "Powered by u003ca class="icon-imgur-white" href="https://imgur.com/"u003eu003c/au003e",
                contentPolicyHtml: "User contributions licensed under u003ca href="https://creativecommons.org/licenses/by-sa/3.0/"u003ecc by-sa 3.0 with attribution requiredu003c/au003e u003ca href="https://stackoverflow.com/legal/content-policy"u003e(content policy)u003c/au003e",
                allowUrls: true
                ,
                noCode: true, onDemand: true,
                discardSelector: ".discard-answer"
                ,immediatelyShowMarkdownHelp:true
                );



                );













                 

                draft saved


                draft discarded


















                StackExchange.ready(
                function ()
                StackExchange.openid.initPostLogin('.new-post-login', 'https%3a%2f%2fmath.stackexchange.com%2fquestions%2f2989273%2fextending-rolles-theorem-to-infinity%23new-answer', 'question_page');

                );

                Post as a guest






























                3 Answers
                3






                active

                oldest

                votes








                3 Answers
                3






                active

                oldest

                votes









                active

                oldest

                votes






                active

                oldest

                votes








                up vote
                4
                down vote













                Consider
                $$
                g(x)=f(tan x)
                $$

                defined over $(-pi/2,pi/2)$. Then we can extend it to $[-pi/2,pi/2]$ by setting
                $$
                g(-pi/2)=g(pi/2)=L
                $$

                The derivative of $g$ exists over $(-pi/2,pi/2)$ and is equal to
                $$
                g'(x)=fracf'(tan x)cos^2x
                $$

                By Rolle's theorem, there exists $c$ such that $g'(c)=0$, so that $f'(tan c)=0$.



                You can do similarly for a function $f$ defined over $[a,infty)$, differentiable over $(a,infty)$ and such that
                $$
                f(a)=lim_xtoinftyf(x)
                $$

                Define $g(x)=f(tan x)$ over $[arctan a,pi/2)$ and the proof will be the same as before.






                share|cite|improve this answer
























                  up vote
                  4
                  down vote













                  Consider
                  $$
                  g(x)=f(tan x)
                  $$

                  defined over $(-pi/2,pi/2)$. Then we can extend it to $[-pi/2,pi/2]$ by setting
                  $$
                  g(-pi/2)=g(pi/2)=L
                  $$

                  The derivative of $g$ exists over $(-pi/2,pi/2)$ and is equal to
                  $$
                  g'(x)=fracf'(tan x)cos^2x
                  $$

                  By Rolle's theorem, there exists $c$ such that $g'(c)=0$, so that $f'(tan c)=0$.



                  You can do similarly for a function $f$ defined over $[a,infty)$, differentiable over $(a,infty)$ and such that
                  $$
                  f(a)=lim_xtoinftyf(x)
                  $$

                  Define $g(x)=f(tan x)$ over $[arctan a,pi/2)$ and the proof will be the same as before.






                  share|cite|improve this answer






















                    up vote
                    4
                    down vote










                    up vote
                    4
                    down vote









                    Consider
                    $$
                    g(x)=f(tan x)
                    $$

                    defined over $(-pi/2,pi/2)$. Then we can extend it to $[-pi/2,pi/2]$ by setting
                    $$
                    g(-pi/2)=g(pi/2)=L
                    $$

                    The derivative of $g$ exists over $(-pi/2,pi/2)$ and is equal to
                    $$
                    g'(x)=fracf'(tan x)cos^2x
                    $$

                    By Rolle's theorem, there exists $c$ such that $g'(c)=0$, so that $f'(tan c)=0$.



                    You can do similarly for a function $f$ defined over $[a,infty)$, differentiable over $(a,infty)$ and such that
                    $$
                    f(a)=lim_xtoinftyf(x)
                    $$

                    Define $g(x)=f(tan x)$ over $[arctan a,pi/2)$ and the proof will be the same as before.






                    share|cite|improve this answer












                    Consider
                    $$
                    g(x)=f(tan x)
                    $$

                    defined over $(-pi/2,pi/2)$. Then we can extend it to $[-pi/2,pi/2]$ by setting
                    $$
                    g(-pi/2)=g(pi/2)=L
                    $$

                    The derivative of $g$ exists over $(-pi/2,pi/2)$ and is equal to
                    $$
                    g'(x)=fracf'(tan x)cos^2x
                    $$

                    By Rolle's theorem, there exists $c$ such that $g'(c)=0$, so that $f'(tan c)=0$.



                    You can do similarly for a function $f$ defined over $[a,infty)$, differentiable over $(a,infty)$ and such that
                    $$
                    f(a)=lim_xtoinftyf(x)
                    $$

                    Define $g(x)=f(tan x)$ over $[arctan a,pi/2)$ and the proof will be the same as before.







                    share|cite|improve this answer












                    share|cite|improve this answer



                    share|cite|improve this answer










                    answered 5 hours ago









                    egreg

                    172k1283194




                    172k1283194




















                        up vote
                        3
                        down vote













                        If $f=L$ everywhere, the conclusion is clear. So let's assume $f(x_0)ne L$ for some $x_0.$ WLOG, $f(x_0)>L.$ Because of the given limit conditions, there exists $a>|x_0|$ such that both $f(-a),f(a) < f(x_0).$ Then the maximum value of $f$ on $[-a,a]$ can't occur at either of the end points. It thus occurs at an interior point $c,$ and at that $c$ we have $ f'(c)=0.$






                        share|cite|improve this answer
























                          up vote
                          3
                          down vote













                          If $f=L$ everywhere, the conclusion is clear. So let's assume $f(x_0)ne L$ for some $x_0.$ WLOG, $f(x_0)>L.$ Because of the given limit conditions, there exists $a>|x_0|$ such that both $f(-a),f(a) < f(x_0).$ Then the maximum value of $f$ on $[-a,a]$ can't occur at either of the end points. It thus occurs at an interior point $c,$ and at that $c$ we have $ f'(c)=0.$






                          share|cite|improve this answer






















                            up vote
                            3
                            down vote










                            up vote
                            3
                            down vote









                            If $f=L$ everywhere, the conclusion is clear. So let's assume $f(x_0)ne L$ for some $x_0.$ WLOG, $f(x_0)>L.$ Because of the given limit conditions, there exists $a>|x_0|$ such that both $f(-a),f(a) < f(x_0).$ Then the maximum value of $f$ on $[-a,a]$ can't occur at either of the end points. It thus occurs at an interior point $c,$ and at that $c$ we have $ f'(c)=0.$






                            share|cite|improve this answer












                            If $f=L$ everywhere, the conclusion is clear. So let's assume $f(x_0)ne L$ for some $x_0.$ WLOG, $f(x_0)>L.$ Because of the given limit conditions, there exists $a>|x_0|$ such that both $f(-a),f(a) < f(x_0).$ Then the maximum value of $f$ on $[-a,a]$ can't occur at either of the end points. It thus occurs at an interior point $c,$ and at that $c$ we have $ f'(c)=0.$







                            share|cite|improve this answer












                            share|cite|improve this answer



                            share|cite|improve this answer










                            answered 4 hours ago









                            zhw.

                            69.9k42974




                            69.9k42974




















                                up vote
                                1
                                down vote













                                Derivatives satisfy the intermediate value property.



                                If $f'(c) not= 0$ for all $c$, then either $f'(c) > 0$ for all $c$ or $f'(c) < 0$ for all $c$.



                                In either case, $f$ is strictly monotone ruling out $displaystyle lim_x to -infty f(x) = lim_x to infty f(x)$.






                                share|cite|improve this answer
























                                  up vote
                                  1
                                  down vote













                                  Derivatives satisfy the intermediate value property.



                                  If $f'(c) not= 0$ for all $c$, then either $f'(c) > 0$ for all $c$ or $f'(c) < 0$ for all $c$.



                                  In either case, $f$ is strictly monotone ruling out $displaystyle lim_x to -infty f(x) = lim_x to infty f(x)$.






                                  share|cite|improve this answer






















                                    up vote
                                    1
                                    down vote










                                    up vote
                                    1
                                    down vote









                                    Derivatives satisfy the intermediate value property.



                                    If $f'(c) not= 0$ for all $c$, then either $f'(c) > 0$ for all $c$ or $f'(c) < 0$ for all $c$.



                                    In either case, $f$ is strictly monotone ruling out $displaystyle lim_x to -infty f(x) = lim_x to infty f(x)$.






                                    share|cite|improve this answer












                                    Derivatives satisfy the intermediate value property.



                                    If $f'(c) not= 0$ for all $c$, then either $f'(c) > 0$ for all $c$ or $f'(c) < 0$ for all $c$.



                                    In either case, $f$ is strictly monotone ruling out $displaystyle lim_x to -infty f(x) = lim_x to infty f(x)$.







                                    share|cite|improve this answer












                                    share|cite|improve this answer



                                    share|cite|improve this answer










                                    answered 4 hours ago









                                    Umberto P.

                                    37.3k12962




                                    37.3k12962



























                                         

                                        draft saved


                                        draft discarded















































                                         


                                        draft saved


                                        draft discarded














                                        StackExchange.ready(
                                        function ()
                                        StackExchange.openid.initPostLogin('.new-post-login', 'https%3a%2f%2fmath.stackexchange.com%2fquestions%2f2989273%2fextending-rolles-theorem-to-infinity%23new-answer', 'question_page');

                                        );

                                        Post as a guest













































































                                        Comments

                                        Popular posts from this blog

                                        Long meetings (6-7 hours a day): Being “babysat” by supervisor

                                        Is the Concept of Multiple Fantasy Races Scientifically Flawed? [closed]

                                        Confectionery